taks test preparation workbook, grade 10 - student edition€¦ · rights was created to address...

87
T E S T T E S T P R A C T I C E T A K S P R A C T I C E T A K S Texas SOCIAL STUDIES GRADE 10

Upload: others

Post on 20-Jun-2020

1 views

Category:

Documents


0 download

TRANSCRIPT

Page 1: TAKS Test Preparation Workbook, Grade 10 - Student Edition€¦ · Rights was created to address colonists’ anger at the British government for— F placing taxes on common goods

TESTTESTPRACTICE

TAKS PRACTICE

TAKSTexas

SOCIAL STUDIESGRADE 10

Page 2: TAKS Test Preparation Workbook, Grade 10 - Student Edition€¦ · Rights was created to address colonists’ anger at the British government for— F placing taxes on common goods

Copyright © Glencoe/McGraw-Hill. All rights reserved. Permission is granted to reproduce material contained herein on thecondition that such material be reproduced only for classroom use; and be provided to students, teachers, and familieswithout charge. Any other reproduction, for use or sale, is prohibited without prior written permission of the publisher.

Send all inquiries to:Glencoe/McGraw-Hill8787 Orion PlaceColumbus, OH 43240-4027

ISBN 0-07-831368-6

Printed in the United States of America

2 3 4 5 6 7 8 9 009 06 05 04 03

Glencoe/McGraw-Hill

This booklet was written by The Princeton Review, one of the nation’s leaders in test preparation. The Princeton Review helps millions of students every year prepare for standardized assessments of all kinds. Through its association withGlencoe/McGraw-Hill, The Princeton Review offers the best way to help students excel on the TAKS.

The Princeton Review is not affiliated with Princeton University or Educational Testing Service.

Page 3: TAKS Test Preparation Workbook, Grade 10 - Student Edition€¦ · Rights was created to address colonists’ anger at the British government for— F placing taxes on common goods

CONTENTSIntroduction . . . . . . . . . . . . . . . . . . . . . . . . . . . . . . . . . . . . . . . . . . . . . . . . . . . . . . . 1

Diagnostic Test . . . . . . . . . . . . . . . . . . . . . . . . . . . . . . . . . . . . . . . . . . . . . . . . . . . . 3

Lesson 1: Using the Process of Elimination . . . . . . . . . . . . . . . . . . . . . . . . . . . . . . 27

Lesson 2: Comparing and Contrasting . . . . . . . . . . . . . . . . . . . . . . . . . . . . . . . . . . 31

Lesson 3: Relating Cause and Effect . . . . . . . . . . . . . . . . . . . . . . . . . . . . . . . . . . . 35

Lesson 4: Putting Events into Sequence . . . . . . . . . . . . . . . . . . . . . . . . . . . . . . . . . 39

Lesson 5: Reading and Interpreting Maps. . . . . . . . . . . . . . . . . . . . . . . . . . . . . . . . 43

Lesson 6: Reading and Interpreting Charts and Tables . . . . . . . . . . . . . . . . . . . . . . 47

Lesson 7: Reading and Interpreting Graphs . . . . . . . . . . . . . . . . . . . . . . . . . . . . . . 51

Lesson 8: Interpreting Quotations. . . . . . . . . . . . . . . . . . . . . . . . . . . . . . . . . . . . . . 55

Lesson 9: Interpreting Illustrations and Political Cartoons . . . . . . . . . . . . . . . . . . . 59

Practice Test. . . . . . . . . . . . . . . . . . . . . . . . . . . . . . . . . . . . . . . . . . . . . . . . . . . . . . 63

Page 4: TAKS Test Preparation Workbook, Grade 10 - Student Edition€¦ · Rights was created to address colonists’ anger at the British government for— F placing taxes on common goods
Page 5: TAKS Test Preparation Workbook, Grade 10 - Student Edition€¦ · Rights was created to address colonists’ anger at the British government for— F placing taxes on common goods

Cop

yrig

ht ©

by T

he M

cGra

w-H

ill C

ompa

nies

, In

c.

Introduction 1

INTRODUCTION ABOUT THIS BOOK

This book is designed to help you practice for the social studies section of the TexasAssessment of Knowledge and Skills (TAKS), a test that you will take in the tenth grade.The book includes the following sections:

1. Diagnostic Test

2. Lessons

3. Practice Test

The Diagnostic Test is similar to the social studies TAKS. It includes forty multiple-choicequestions with four answer choices for each question. The purpose of the Diagnostic Testis to identify your strengths and weaknesses early in the school year so that you canreview the skills you need to improve your score on the TAKS.

The nine Lessons will help you review specific skills you need to do well on the socialstudies TAKS. Each lesson includes two sections: Learn It and Use It. The Learn It sectionwill help you answer multiple-choice questions by guiding you through a step-by-stepprocess. The Use It section will give you practice answering multiple-choice questions onyour own.

The Practice Test is similar to the social studies TAKS. It includes forty multiple-choicequestions with four answer choices for each question. You will take this test to apply whatyou have learned in this book and in your tenth-grade social studies class. After you takethe Practice Test, you will have an idea of how much you improved since the time youtook the Diagnostic Test.

THE TEXAS ASSESSMENT OF KNOWLEDGE AND SKILLS

The TAKS is a standardized test that is intended to show what you have been learning inyour classes. In grade ten, the social studies section of the test is administered along withsections in English language arts, mathematics, and science.

Most of the questions on the social studies TAKS will ask about U.S. history and thegeographic, economic, cultural, and political influences on it. Keep in mind that there willalso be some questions that test your general knowledge of world history and worldgeography. These questions will often ask you to use social studies skills, such asreading and interpreting information on maps, charts, or graphs.

The social studies TAKS is a multiple-choice test. Each question on the test will requireyou to select the correct answer from four answer choices. Because all of the questionswill be scored, you should not leave any of the questions unanswered. If possible, do nottake a wild guess. Use the process of elimination to rule out as many answer choices asyou can, and pick one of the remaining choices. You will learn more about using theprocess of elimination in the first lesson in this book.

Page 6: TAKS Test Preparation Workbook, Grade 10 - Student Edition€¦ · Rights was created to address colonists’ anger at the British government for— F placing taxes on common goods

2 TAKS Test Practice Workbook, Social Studies Grade 10

Copyright ©

by The M

cGraw

-Hill C

ompanies, Inc.

TEST TIPS AND TECHNIQUES

Try to apply these strategies as you work through the lessons and tests in this book. Getin the habit of using them. They should be second nature by the time you take the TAKS.

• Pace yourself. If you work too quickly, you are more likely to make carelessmistakes. Instead, pace yourself by working carefully through each question. Whenthere is a question that you cannot answer, do not get stuck on it and waste valuabletime. Skip it and come back to answer it at the end. It is better to move on to otherquestions that you might find easier.

• Read questions carefully. It is easy to make careless mistakes if you read tooquickly.

• Be systematic. Some of the lessons in this book suggest steps for you to take sothat you can answer the questions correctly. Think about the steps you use to answereach question in the book, not just the questions that come with suggested steps.

• Make educated guesses. When you reach the end of the test, be sure to go backand answer the questions that you skipped the first time. Any question left blank willbe marked wrong, so it is better to take your best guess and have a chance of gettingthe question correct than to leave it blank and guarantee that you will get it wrong.Remember to always use the process of elimination to narrow the possibilities. Thenmake your guess from the remaining choices.

• Check your work. If you finish the test with time to spare, use the extra time tocheck your work. You do not get any extra points for finishing early, so it is better tocheck over as many of your answers as you can. Make sure you did not make anysilly mistakes!

• Be comfortable. It is important to feel rested and comfortable on test day in orderto do your best. Try to get plenty of sleep for several nights before the test. Makesure to eat a good breakfast and lunch on test day. Be prepared: bring severalpencils, a sharpener, a watch, and tissues (if you need them). Use the bathroomjust before test time.

• Stay positive. Realize that you will encounter some questions that you will finddifficult—that is natural. Keep a positive attitude while you work through the book,and remind yourself that practice makes perfect.

Page 7: TAKS Test Preparation Workbook, Grade 10 - Student Edition€¦ · Rights was created to address colonists’ anger at the British government for— F placing taxes on common goods

Diagnostic Test

Page 8: TAKS Test Preparation Workbook, Grade 10 - Student Edition€¦ · Rights was created to address colonists’ anger at the British government for— F placing taxes on common goods
Page 9: TAKS Test Preparation Workbook, Grade 10 - Student Edition€¦ · Rights was created to address colonists’ anger at the British government for— F placing taxes on common goods

DIRECTIONS

Read each question and choose the bestanswer. Then mark the answer you havechosen.

SAMPLE A

Under the Constitution, the president hasthe power to—

A use executive privilege to waivefederal laws

B hold jury trials

C pardon citizens convicted of crimes

D fill vacant governor seats

Cop

yrig

ht ©

by T

he M

cGra

w-H

ill C

ompa

nies

, In

c.

Diagnostic Test 5

Page 10: TAKS Test Preparation Workbook, Grade 10 - Student Edition€¦ · Rights was created to address colonists’ anger at the British government for— F placing taxes on common goods

1 Which of the following events took placein 1776?

A The Civil War broke out.

B The Boston Massacre occurred.

C George Washington became thefirst president.

D The Declaration of Independencewas signed.

2 At the first meeting of the U.S. Congress,the Bill of Rights was added to theConstitution primarily to—

F protect the individual rights of theAmerican people

G divide the government into threebranches to limit its power

H ensure that small states possessedas much power as big states

J safeguard the rights of Southernslaveholders

3 The invention of new machinery, such asthe steam-powered loom, and new powersources (such as steam) primarily led to—

A factories that did not need to be builtnear streams and rivers

B higher wages for factory workers

C more artisans who could producecotton from their own homes

D more expensive clothing

4 The primary goal of subsistence farmers,as opposed to commercial farmers, is to—

F supplement their income by growingextra food

G grow crops solely to sell to others ata profit

H produce enough food to feedthemselves

J gather food from plants that growin the wild

6 TAKS Test Practice Workbook, Social Studies Grade 10

Copyright ©

by The M

cGraw

-Hill C

ompanies, Inc.

Page 11: TAKS Test Preparation Workbook, Grade 10 - Student Edition€¦ · Rights was created to address colonists’ anger at the British government for— F placing taxes on common goods

Use the information in the box and yourknowledge of social studies to answer question 5.

5 These are the main principles of—

A separation of power

B popular sovereignty

C checks and balances

D Federalism

6 The Third Amendment of the Bill ofRights was created to address colonists’anger at the British government for—

F placing taxes on common goods

G housing military troops in privatehomes

H restricting freedom of speech

J mandating a national religion

1. Individual states have thepower to oversee tradewithin their borders.

2. The United States has thepower to declare war.

3. If national laws and statelaws contradict each other,the national laws must beobeyed.

Cop

yrig

ht ©

by T

he M

cGra

w-H

ill C

ompa

nies

, In

c.

Diagnostic Test 7

Page 12: TAKS Test Preparation Workbook, Grade 10 - Student Edition€¦ · Rights was created to address colonists’ anger at the British government for— F placing taxes on common goods

8 TAKS Test Practice Workbook, Social Studies Grade 10

Copyright ©

by The M

cGraw

-Hill C

ompanies, Inc.

7 From the graph it can be concluded that—

A Rwanda has more acres of land setaside for wildlife preserves thanKenya

B there is equal support in Uganda andZambia for creating parks

C South Africa has a smaller percentageof its land set aside than Tanzania

D Botswana has the largest parks of allthe countries listed

Use the graph and your knowledge of social studies to answer question 7.

Percentage of Total Land Set Aside for Wildlife Reserves

0 2 4 6 8 10 12 14 16

Botswana

KenyaRwanda

South AfricaTanzaniaUgandaZambia

Zimbabwe

DemocraticRepublic ofthe Congo

Percent

Co

un

trie

s

Page 13: TAKS Test Preparation Workbook, Grade 10 - Student Edition€¦ · Rights was created to address colonists’ anger at the British government for— F placing taxes on common goods

8 This quotation shows that Abigail Adamsbelieved that—

F the new constitution should providewomen with greater rights

G women should have the right to ownproperty

H all men act like tyrants in theirtreatment of women

J the right to vote and run for electedoffice should be offered to women

Cop

yrig

ht ©

by T

he M

cGra

w-H

ill C

ompa

nies

, In

c.

Diagnostic Test 9

Use the quotation and your knowledge of social studies to answer question 8.

In the new Code of Laws which I suppose it will be necessary for you to make I would desire youwould Remember the Ladies, and be more generous and favorable to them than your ancestors. Do notput such unlimited power into the hands of the Husbands. Remember all Men would be tyrants if theycould. If particular care and attention is not paid to the Ladies we are determined to foment a Rebellion,and will not hold ourselves bound by any Laws in which we have no voice, or Representation.

—Abigail Adams, 1776, from a letter to her husband, John Adams

Page 14: TAKS Test Preparation Workbook, Grade 10 - Student Edition€¦ · Rights was created to address colonists’ anger at the British government for— F placing taxes on common goods

9 England’s Magna Carta provided theFramers of the U.S. Constitution with whichof the following important principles?

A People’s right to life, liberty, andproperty

B Dividing the government into threebranches

C Limiting a ruler’s power

D Forming a union for defense purposes

10 Manifest Destiny, a term used to describethe United States’s eagerness to claim theentire continent for itself during the mid-1800s, led to—

F a stronger federal government to rulethe new lands

G conflicts with both Mexicans andNative Americans

H renewed conflict with France andBritain

J agreement between antislavery andpro-slavery states

11 The Fugitive Slave Act of 1850 wasdesigned to protect the institution ofslavery by imposing stiffer penalties onthose who aided runaway slaves, but italso—

A stopped the Underground Railroadbecause it was too dangerous

B led to increased educationalopportunities for slaves

C increased the price of slaves so thatmany people could not afford to buythem

D outraged many people and increasedsupport for the end of slavery

12 One of the most significant flaws in theArticles of Confederation, which wereratified in 1781, was that—

F Congress did not have the power tocreate an army

G Congress could not enforce any of thelaws that it passed

H the power of the states was weakenedsignificantly

J small states were not allowed to sendas many delegates as big states

10 TAKS Test Practice Workbook, Social Studies Grade 10

Copyright ©

by The M

cGraw

-Hill C

ompanies, Inc.

Page 15: TAKS Test Preparation Workbook, Grade 10 - Student Edition€¦ · Rights was created to address colonists’ anger at the British government for— F placing taxes on common goods

13 During the 1830s and 1840s, two menindependently invented sewing machines.This invention led to the mass productionof clothing and the rise of factories andeventually contributed to—

A an increase in the number of menworking in factories

B better relationships between factoryowners and their workers

C the rise of labor unions to address lowwages and bad working conditions

D shorter workdays for children underthe age of twelve

Use the map and your knowledge of socialstudies to answer question 14.

14 Based on the map, you can conclude thatmost cities with large populations arelocated—

F in South America

G on the African continent

H in the Northern Hemisphere

J near large bodies of water

City with a large population

Cop

yrig

ht ©

by T

he M

cGra

w-H

ill C

ompa

nies

, In

c.

Diagnostic Test 11

Page 16: TAKS Test Preparation Workbook, Grade 10 - Student Edition€¦ · Rights was created to address colonists’ anger at the British government for— F placing taxes on common goods

15 The Coercive Acts, harsh measurespassed by the British Parliament to punishcolonists for their involvement in theBoston Tea Party, served as a turning pointin colonial affairs and almost immediatelyled to—

A the colony of Massachusettsapologizing for the crime and offeringto pay for the destroyed tea

B the passage of the Quebec Act, whichincreased the size of the province ofQuebec

C the formation of the First ContinentalCongress and the colonists’ demandsfor such rights as “life, liberty, andproperty”

D the Olive Branch Petition, the colonists’last attempt to reach a compromise withBritain and avoid war

16 The invention of the cotton gin in 1793revolutionized the process of preparingcotton. This technological breakthroughalso led to—

F a sharp decline in the need for thelabor of enslaved people

G an increase in demand for other cropsgrown in the South

H the spread of cotton plantationsfarther south and west

J a slowdown at textile factories in theNorthern states

17 In 1845, more than forty thousand Irishimmigrants came to the United States,largely due to the fact that—

A the U.S. government offered highsalaries to Irish workers

B a plant disease had wiped out a largeportion of the potato crop in Ireland

C in that year, an innovation inshipbuilding made travel overseas farsafer

D Ireland had experienced a hugepopulation growth in 1844

12 TAKS Test Practice Workbook, Social Studies Grade 10

Copyright ©

by The M

cGraw

-Hill C

ompanies, Inc.

Page 17: TAKS Test Preparation Workbook, Grade 10 - Student Edition€¦ · Rights was created to address colonists’ anger at the British government for— F placing taxes on common goods

18 According to the chart, which of thesestatements is true?

F The invention of new technologieswas dependent on private investment.

G The textile industry benefited fromsome inventions of the 1700s and1800s.

H Transportation inventions primarilybenefited those in the United States.

J Advances in technology improvedpeople’s quality of life.

Cop

yrig

ht ©

by T

he M

cGra

w-H

ill C

ompa

nies

, In

c.

Diagnostic Test 13

Use the chart and your knowledge of social studies to answer question 18.

Important Inventions

1698173317641782179318041851

Thomas SaveryJohn Kay

John HargreavesJames WattEli Whitney

Richard TrevithickIsaac Singer

Water pumpFlying shuttle

Spinning jennyRotary steam engine

Cotton ginSteam train

Sewing machine

Year Inventor Invention

Page 18: TAKS Test Preparation Workbook, Grade 10 - Student Edition€¦ · Rights was created to address colonists’ anger at the British government for— F placing taxes on common goods

19 Based on the map, which of the followingconclusions is true?

A The majority of towns and cities arelocated in the regions that were settledearliest.

B Jamestown is the only Southern citythat was established before 1660.

C The Middle Colonies consisted ofonly two colonies.

D No one lived in North Carolinabefore 1760.

14 TAKS Test Practice Workbook, Social Studies Grade 10

Copyright ©

by The M

cGraw

-Hill C

ompanies, Inc.

Use the map and your knowledge of social studies to answer question 19.

Settlement Patterns Between 1607 and 1760

Atlantic Ocean

Before 1660Between 1660 and 1700Between 1700 and 1760Regional boundaryTown or city

200 kilometers

100100

0

0

Savannah

Georgia

Pennsylvania

SouthernColonies

MiddleColonies

Jamestown

St. Mary’s

Delaware

Philadelphia

N.J. New YorkNew Haven

R.I.ProvidenceConn.

PlymouthBoston

SalemMass.

NewYorkHartford

N.H.

NewEngland

FRENCHCANADA

Lake Erie

Lake Ontario

Virginia

SouthCarolina

North Carolina

Md.

SPANISHFLORIDA

200 miles

Page 19: TAKS Test Preparation Workbook, Grade 10 - Student Edition€¦ · Rights was created to address colonists’ anger at the British government for— F placing taxes on common goods

20 In the mid-1700s, the Southern Colonieshad more enslaved African Americans thaneither New England or the MiddleColonies. This fact can be attributed to—

F the Southern Colonies’ proximity toWest Africa, where most slavescame from

G the aristocratic background of thecolonists and their need for householdservants

H the climate of the Southern Colonies,which was perfect for growing labor-intensive crops

J the rapid growth of factories and theneed for workers to run them

21 The main purpose of the Declaration ofIndependence was to—

A clarify the rules of the Britishgovernment

B establish the United States as a nation

C list grievances against King GeorgeIII

D nominate Thomas Jefferson forpresident

Cop

yrig

ht ©

by T

he M

cGra

w-H

ill C

ompa

nies

, In

c.

Diagnostic Test 15

Page 20: TAKS Test Preparation Workbook, Grade 10 - Student Edition€¦ · Rights was created to address colonists’ anger at the British government for— F placing taxes on common goods

Use the map and your knowledge of socialstudies to answer question 22.

22 According to the map, which of thefollowing statements is true?

F The coast of Peru is humid andsubtropical.

G Colombia is covered by a large desert.

H Southern Argentina receives a lot ofrain.

J Brazil has a mostly tropical climate.

Use the graphs and your knowledge of socialstudies to answer question 23.

23 Which of the following conclusions do thedata on the graphs above support?

A The population of the South waslarger than the population of theNorth.

B The North was more industrializedthan the South.

C The South had more farmland than theNorth.

D Northerners trusted the banks morethan Southerners.

Population

Bank deposits

Farmland

Manufacturedgoods

Resources of theNorth and South, 1861

29%

19%

35%

8%

71%

92%

65%

81%

(1/3 enslaved)

North SouthTropical rain forestTropical savanna

Steppe

MediterraneanHumid subtropical

Marine west coastHighlands

Desert

Tropical Dry

Mid-Latitude

16 TAKS Test Practice Workbook, Social Studies Grade 10

Copyright ©

by The M

cGraw

-Hill C

ompanies, Inc.

Page 21: TAKS Test Preparation Workbook, Grade 10 - Student Edition€¦ · Rights was created to address colonists’ anger at the British government for— F placing taxes on common goods

24 The use of steam instead of water tooperate factories led to—

F higher prices for manufactured goods

G the spread of factories throughoutthe North

H lower demand for skilled workers

J the beginning of industrial growthin the South

25 How did the discovery of gold inCalifornia affect Native Americans?

A New educational opportunitiesbecame available to them.

B They were forced to migrate north toescape the new settlers.

C Native Americans staked their ownclaims and grew wealthy.

D The population of Native Americansdeclined dramatically.

26 Impeachment cases for federal officialsaccused of unlawful activity are triedby the—

F president

G House of Representatives

H Senate

J secretary of state

27 Which one of the following ideas iscommon to both the U.S. Bill of Rightsand the English Bill of Rights?

A Restriction on the housing of soldiersin citizens’ homes

B Protection from involuntary searchand seizure

C Trial by jury for all citizens accusedof a crime

D Limitations of the power of thefederal government

Cop

yrig

ht ©

by T

he M

cGra

w-H

ill C

ompa

nies

, In

c.

Diagnostic Test 17

Page 22: TAKS Test Preparation Workbook, Grade 10 - Student Edition€¦ · Rights was created to address colonists’ anger at the British government for— F placing taxes on common goods

Use the map and your knowledge of socialstudies to answer question 28.

28 According to the Compromise of 1850,which area contained the majority of slavestates?

F A

G B

H C

J D

29 The “unalienable rights” in theDeclaration of Independence refer to—

A the right to bear arms

B the ability to trade with othercountries

C life, liberty, and the pursuit ofhappiness

D the necessity of having arepresentative government

30 What was added to the U.S. Constitutionto guarantee individual rights?

F The Bill of Rights

G The Declaration of Independence

H The Preamble

J Constitutional restrictions

AB

C D

18 TAKS Test Practice Workbook, Social Studies Grade 10

Copyright ©

by The M

cGraw

-Hill C

ompanies, Inc.

Page 23: TAKS Test Preparation Workbook, Grade 10 - Student Edition€¦ · Rights was created to address colonists’ anger at the British government for— F placing taxes on common goods

31 Which of the following conclusions doesthe chart support?

A The standard of living is lower inPapua New Guinea than in Australia.

B Australia has a larger population thanPapua New Guinea.

C Fewer people receive medical care inPapua New Guinea than in Australia.

D Papua New Guinea is more isolatedthan Australia.

Cop

yrig

ht ©

by T

he M

cGra

w-H

ill C

ompa

nies

, In

c.

Diagnostic Test 19

Use the chart and your knowledge of social studies to answer question 31.

Comparative Data forAustralia and Papua New Guinea

Life Expectancy

Infant Mortality Rate(per 1,000 births)

Literacy Rate

Australia

56 years78 years

PapuaNew GuineaCountry

100%

6 77

72%

Page 24: TAKS Test Preparation Workbook, Grade 10 - Student Edition€¦ · Rights was created to address colonists’ anger at the British government for— F placing taxes on common goods

32 Why did the British Parliament allow theEast India Company to maintain amonopoly on tea when the colonistsargued against it?

F To provoke the colonists to split fromBritish rule

G So the British could concentrate onother businesses

H So the company could avoidbankruptcy

J To assert authority over the colonists

33 The nullification crisis in 1832 testedwhich American principle?

A Federalism

B Separation of powers

C Republicanism

D Individual rights

34 The differences between commercialindustries and cottage industries is bestcharacterized in terms of—

F the quality of the goods produced

G scale and location of the business

H the types of goods produced

J workers’ level of education

35 The Thirteenth, Fourteenth, and FifteenthAmendments were important becausethey—

A provided African Americans withaccess to public education

B ended slavery and extended votingrights to African American men

C extended full voting rights to women

D offered concessions to the South inhopes of reuniting the country

20 TAKS Test Practice Workbook, Social Studies Grade 10

Copyright ©

by The M

cGraw

-Hill C

ompanies, Inc.

Page 25: TAKS Test Preparation Workbook, Grade 10 - Student Edition€¦ · Rights was created to address colonists’ anger at the British government for— F placing taxes on common goods

36 Using the map, what inferences can bedrawn about settlement patterns inAustralia?

F People congregated along the GreatDividing Range.

G People settled near major plains andplateaus.

H People settled mostly in westernAustralia.

J People did not settle in the centraldesert of Australia.

Cop

yrig

ht ©

by T

he M

cGra

w-H

ill C

ompa

nies

, In

c.

Diagnostic Test 21

Use the map and your knowledge of social studies to answer question 36.

IndianOcean

TasmanSea

CoralSea

IndianOcean

Gulf ofCarpentaria

Arafura Sea

Western Australia

Northern Territory

South Australia

Queensland

New South Wales

Victoria

Tasmania

Darwin

Perth

Cairns

Townsville

Brisbane

BourkeBroken Hill

NewcastleSydney

Melbourne

Whylla

Great AustralianBight

Nullarbor Plain

Great Dividing

Range

KimberlyPlateau

Arnhem Land

GreatSandyDesert

GreatVictoriaDesert

GibsonDesert

Western Plateau

GreatArtesian

Basin

BarklyTableland

Hammersly Range

Austra

lian

Alps

CanberraAdelaide

AU S T R A L I A

Page 26: TAKS Test Preparation Workbook, Grade 10 - Student Edition€¦ · Rights was created to address colonists’ anger at the British government for— F placing taxes on common goods

Use the quotation and your knowledge ofsocial studies to answer question 37.

37 Based upon the quotation, the speaker’sattitude toward the institution of slaverycould best be described as—

A nostalgic

B critical

C nonjudgmental

D ambivalent

38 The development of clipper shipsimproved trade with China by—

F allowing merchants to carry biggerloads

G dramatically shortening the round-tripjourney time

H decreasing the number of sailorsneeded for the voyage

J offering better protection againstpirates

They would stand the slaves up on theblock and talk about what a fine-looking specimen . . . they was, tellhow healthy they was, look in theirmouth and examine their teeth just likethey was a horse, and talk about thekind of work they would be fit for andcould do.

—A formerly enslaved person’sdescription of slave auctions

22 TAKS Test Practice Workbook, Social Studies Grade 10

Copyright ©

by The M

cGraw

-Hill C

ompanies, Inc.

Page 27: TAKS Test Preparation Workbook, Grade 10 - Student Edition€¦ · Rights was created to address colonists’ anger at the British government for— F placing taxes on common goods

39 What can you infer about the route of theOregon Trail based upon the map?

A It followed the path of major rivers.

B It cut through the lowest part of theRocky Mountains.

C The most difficult part of the journeywas through the Utah Territory.

D It gradually traveled south fromIndependence, Missouri.

Cop

yrig

ht ©

by T

he M

cGra

w-H

ill C

ompa

nies

, In

c.

Diagnostic Test 23

Use the map and your knowledge of social studies to answer question 39.

The Oregon and California Trails

San Francisco

Independence

UtahTerritory

New MexicoTerritory

MinnesotaTerritory

UnorganizedTerritory

Texas

OregonTerritory

SaltLakeCity

GreatSaltLake

California TrailOregon TrailRiver

Rocky MountainsC

asca

des

Sierra

Nevada

C

o lumbia R.Missouri R.

Yellowstone R.

North Pla tte R.

S

outh

Platte R.

Rio

Grande

Arkans as R.

Snake

R

.

California

Sacramento

Portland

Page 28: TAKS Test Preparation Workbook, Grade 10 - Student Edition€¦ · Rights was created to address colonists’ anger at the British government for— F placing taxes on common goods

40 How did the colonists protest both theStamp Act and the Townshend Acts?

F They initiated the Boston Tea Party.

G They boycotted British goods.

H They created a flag to signify theirindependence.

J They created their own system oftaxation.

24 TAKS Test Practice Workbook, Social Studies Grade 10

Copyright ©

by The M

cGraw

-Hill C

ompanies, Inc.

Page 29: TAKS Test Preparation Workbook, Grade 10 - Student Edition€¦ · Rights was created to address colonists’ anger at the British government for— F placing taxes on common goods
Page 30: TAKS Test Preparation Workbook, Grade 10 - Student Edition€¦ · Rights was created to address colonists’ anger at the British government for— F placing taxes on common goods
Page 31: TAKS Test Preparation Workbook, Grade 10 - Student Edition€¦ · Rights was created to address colonists’ anger at the British government for— F placing taxes on common goods

Learn ItUsing the Process of Elimination

Many of the questions on the TAKS may ask you to recall facts and definitions.Eliminating wrong answer choices can help you to answer these questions. Usethe process of elimination when you do not know the correct answer to amultiple-choice question. First, rule out answer choices you know are wrong.Then, choose the best answer from the answer choices that remain. Learn howto use the process of elimination by completing the following practice question.

1� The Framers of the U.S. Constitution gave the president the right to veto billspassed by Congress as part of their effort to—

A create and foster a system of direct democracy

B ensure that the U.S. president had unlimited power

C create a system of checks and balances in the federal government

D design a Federalist system of government

You may not know what motivated the Framers of the U.S. Constitution togive the president the right to veto bills passed by Congress, but you mayknow which answer choices do not fully explain their motivation. Is answerchoice (A) correct?

Yes, it is correct. I’ll read the other answer choices just in case.

No, it is not correct because .

Eliminate it.

I don’t know. I’ll keep it and read the other answerchoices.

Step1

Lesson

1

Test Tip

Use the process ofelimination on everymultiple-choice questionthat you don’t know theanswer to right away.Even if you can get ridof only one answerchoice, you will stillsignificantly increaseyour chances of findingthe right answer.

Lessons 27

Cop

yrig

ht ©

by T

he M

cGra

w-H

ill C

ompa

nies

, In

c.

Lessons 27

Page 32: TAKS Test Preparation Workbook, Grade 10 - Student Edition€¦ · Rights was created to address colonists’ anger at the British government for— F placing taxes on common goods

28 TAKS Test Practice Workbook, Social Studies Grade 10

Copyright ©

by The M

cGraw

-Hill C

ompanies, Inc.

Is answer choice (B) correct?

Yes, it is correct. I’ll read the other answer choices just in case.

No, it is not correct because .

Eliminate it.

I don’t know. I’ll keep it and read the other answer choices.

Is answer choice (C) correct?

Yes, it is correct. I’ll read the other answer choices just in case.

No, it is not correct because .

Eliminate it.

I don’t know. I’ll keep it and read the other answer choices.

Is answer choice (D) correct?

Yes, it is correct. I’ll read the other answer choices just in case.

No, it is not correct because .

Eliminate it.

I don’t know. I’ll keep it and read the other answer choices.

Which answer choices remain?

If more than one answer choice remains, try to remember how the nationalgovernment works. Think about the three branches of government (the executivebranch, the judicial branch, and the legislative branch) and how they interact withone another. Choose the most logical answer choice and briefly explain why youchose it.

Step5

Step4

Step3

Step2

Lesson 1: Using the Process of Elimination

Page 33: TAKS Test Preparation Workbook, Grade 10 - Student Edition€¦ · Rights was created to address colonists’ anger at the British government for— F placing taxes on common goods

Cop

yrig

ht ©

by T

he M

cGra

w-H

ill C

ompa

nies

, In

c.

Lessons 29

Use ItLesson 1: Using the Process of Elimination

DIRECTIONS

Read each question carefully and then choosethe best answer.

1 Frederick Douglass played an importantrole in American history by—

A negotiating a treaty between NativeAmericans and colonists

B helping factory workers form laborunions

C fighting for the rights of women

D increasing support for the abolitionistmovement

2 Horace Mann is known for doing which ofthe following?

F He played an instrumental role in theabolitionist movement.

G He helped reform the system ofpunishing criminals.

H He led a campaign to offer free,public education.

J He opened a school for AfricanAmerican children.

3 The Second Great Awakening wascharacterized by—

A an increase in religious faith

B the persecution of fringe groups

C important scientific advances

D literary interest in the American past

4 The passage of the Stamp Act in 1765placed taxes on everyday items such asplaying cards and newspapers. This newtax angered colonists because—

F they thought it was unfair to placesuch high taxes on everyday items

G it was passed by Parliament, and theAmerican colonies had norepresentation in Parliament

H Great Britain was not investing anymoney in the American colonies toensure their welfare

J the American colonial governmentswere already sending money to GreatBritain

Page 34: TAKS Test Preparation Workbook, Grade 10 - Student Edition€¦ · Rights was created to address colonists’ anger at the British government for— F placing taxes on common goods

Copyright ©

by The M

cGraw

-Hill C

ompanies, Inc.

5 Which of the following was acontroversial factor in the decision toadmit Missouri into the Union in 1819?

A Missouri settlers wanted to enter theUnion as a slave state.

B Free states outnumbered slave statesin 1819.

C Maine also wanted to be admitted tothe Union as a slave state.

D Northern states had a smallerpopulation than Southern states.

6 During the American Revolution, theLoyalists supported—

F maintaining ties with Great Britain

G fighting for independence from GreatBritain

H the establishment of the ContinentalNavy

J naming George Washington head ofthe Continental Army

7 Which of the following amendmentswould protect the Ku Klux Klan’s rightto hold a rally in a public place?

A The First Amendment, whichguarantees freedom of religion,speech, press, and assembly

B The Second Amendment, whichallows people to bear arms

C The Fifth Amendment, which protectsthe rights of people accused of crimes

D The Tenth Amendment, which limitsthe power of the federal governmentand protects individuals’ rights

8 Great Britain recognized the United Statesas an independent country and removed itstroops from U.S. lands through the—

F Olive Branch Petition

G Treaty of Paris

H Land Ordinance of 1785

J Declaration of Independence

Lesson 1: Using the Process of Elimination

30 TAKS Test Practice Workbook, Social Studies Grade 10

Page 35: TAKS Test Preparation Workbook, Grade 10 - Student Edition€¦ · Rights was created to address colonists’ anger at the British government for— F placing taxes on common goods

Some questions on the TAKS may ask you to identify the similarities ordifferences between two or more items by categorizing them, or naming anidea or theme that they represent. Learn how to answer these questions bycompleting the following practice question.

1� Federalists coined the term “Anti-Federalists” for people who—

A favored the plan of government outlined in the Constitution

B opposed the ratification of the Constitution

C believed in having a strong central government

D wanted to remain loyal to Great Britain

This question is asking about the differences between two groups whowere involved in ratifying the Constitution. Briefly write what you knowabout each group.

Federalists:

Anti-Federalists:

Step1

Lessons 31

Learn It Lesson

2

Lessons 31

Cop

yrig

ht ©

by T

he M

cGra

w-H

ill C

ompa

nies

, In

c.

Test Tip

It is important to readevery answer choicebefore you choose one.Even though you maythink you have found agood answer, do notassume it is correctuntil you have looked atall of the choices.

Comparing and Contrasting

Page 36: TAKS Test Preparation Workbook, Grade 10 - Student Edition€¦ · Rights was created to address colonists’ anger at the British government for— F placing taxes on common goods

32 TAKS Test Practice Workbook, Social Studies Grade 10

Copyright ©

by The M

cGraw

-Hill C

ompanies, Inc.

Did the Anti-Federalists favor the plan of government outlined in the Constitution?If not, eliminate answer choice (A).

Did the Anti-Federalists oppose the ratification of the Constitution? If not, eliminateanswer choice (B).

Did the Anti-Federalists believe in having a strong central government? If not,eliminate answer choice (C).

Did the Anti-Federalists want to remain loyal to Great Britain? If not, eliminateanswer choice (D).

Look at the remaining answer choices. Based on what you wrote in step 1, choosethe most logical answer choice and briefly explain why you chose it.

Step6

Step5

Step4

Step3

Step2

Lesson 2: Comparing and Contrasting

Page 37: TAKS Test Preparation Workbook, Grade 10 - Student Edition€¦ · Rights was created to address colonists’ anger at the British government for— F placing taxes on common goods

Cop

yrig

ht ©

by T

he M

cGra

w-H

ill C

ompa

nies

, In

c.

Lessons 33

Use ItLesson 2: Comparing and Contrasting

DIRECTIONS

Read each question carefully and then choosethe best answer.

1 What was the most important differencebetween the Federalists and theDemocratic-Republicans, the first twopolitical parties to form in the UnitedStates?

A The Federalists were led by ThomasJefferson, while the Democratic-Republicans were led by AlexanderHamilton.

B The Federalists wanted to reunitewith Great Britain, but theDemocratic-Republicans wanted tostay independent.

C The Federalist Party drew more of itssupport from ordinary Southernersthan did the Democratic-RepublicanParty.

D The Federalists favored a strongfederal government more than theDemocratic-Republicans.

Use the information in the box and yourknowledge of social studies to answer question 2.

2 The inventors Robert Fulton and JohnGriffiths share the distinction of—

F increasing the speed of transportation

G using steam in their inventions

H having only a high school education

J helping farmers increase theirproductivity

3 Which of the following is an importantdifference between the Virginia Plan andthe New Jersey Plan for the Constitution?

A The Virginia Plan weakened thecentral government more than theNew Jersey Plan.

B The Virginia Plan favored large states,while the New Jersey Plan wasadvantageous to small states.

C The Virginia Plan called for a singlelegislative body, while the New JerseyPlan called for a two-house legislature.

D Under the Virginia Plan, the federalgovernment would be divided intothree branches, but under the NewJersey Plan, it would be divided intoonly two.

Early American Inventors

John Deere Lightweight steel plow

Robert Fulton Steamboat

John Griffiths Clipper sailing ship

Walter Hunt Sewing machine

Cyrus McCormick Reaper

Samuel F.B. Morse Telegraph

Page 38: TAKS Test Preparation Workbook, Grade 10 - Student Edition€¦ · Rights was created to address colonists’ anger at the British government for— F placing taxes on common goods

34 TAKS Test Practice Workbook, Social Studies Grade 10

Copyright ©

by The M

cGraw

-Hill C

ompanies, Inc.

Use the table and your knowledge of socialstudies to answer question 4.

4 Which of the following titles bestintroduces the table?

F Colonizing the Americas

G American Rebellion Against Britain

H British Aggression Against theColonists

J Leading Up to the AmericanRevolution

5 Alexander Hamilton’s belief that thegovernment should be involved inpeople’s economic affairs differed fromThomas Jefferson’s belief in—

A laissez-faire economics

B socialism

C government assistance for the poor

D perfect competition

Use the information in the box and yourknowledge of social studies to answer question 6.

6 What is the best title for this list?

F Military Leaders of the RevolutionaryWar

G Generals of the French and IndianWar

H American War Heroes

J Leaders During the Civil War

7 The House of Representatives initiates theimpeachment of political officials, butwho conducts the actual impeachmenttrial?

A The Supreme Court

B The Senate

C The president

D The cabinet

8 Which of the following inventions of theIndustrial Revolution led to the greatestincrease in product manufacturing?

F Interchangeable parts

G Steamboat

H Telegraph

J Incandescent light

Charles Cornwallis—Battle at Yorktown

Daniel Morgan—Battle at Cowpens

William Prescott—Battle of Bunker Hill

Barry St. Leger—Battle of Saratoga

17651773

1774

1775

Parliament passes the Stamp ActColonists dump tea in Boston HarborParliament punishes the colonists by

passing the Coercive ActsBritish and American soldiers exchange

shots at Lexington and Concord

Date Event

Lesson 2: Comparing and Contrasting

Page 39: TAKS Test Preparation Workbook, Grade 10 - Student Edition€¦ · Rights was created to address colonists’ anger at the British government for— F placing taxes on common goods

Some questions on the TAKS may ask you to identify cause-and-effectrelationships. Learn how to answer these questions by completing the followingpractice questions.

1� What possible outcome did Thomas Jefferson hope to avoid when he negotiatedthe Louisiana Purchase?

A A centralized government

B A French empire in the United States

C Spanish acquisition of the land

D A limit on U.S. boundaries

On the lines below, briefly describe what you know about the LouisianaPurchase.

Consider all of the answer choices. Are there any that canbe eliminated immediately based on your knowledge ofthe Louisiana Purchase? If so, briefly explain why thosechoices are incorrect.

Look at the remaining answer choices. Based on whatyou wrote in step 1, choose the most logical answerchoice and briefly explain why you chose it.

Step3

Step2

Step1

Learn It Lesson

3

Cop

yrig

ht ©

by T

he M

cGra

w-H

ill C

ompa

nies

, In

c.

Lessons 35Lessons 35Lessons 35

Test Tip

After reading aquestion, answer it inyour own words beforeyou read the answerchoices. When youread all of the choices,you may find one thatis similar to the oneyou thought of onyour own!

Relating Cause and Effect

Page 40: TAKS Test Preparation Workbook, Grade 10 - Student Edition€¦ · Rights was created to address colonists’ anger at the British government for— F placing taxes on common goods

Copyright ©

by The M

cGraw

-Hill C

ompanies, Inc.

Now try another example of a cause-and-effect question.

2� Which of the following was a result of the invention of the spinning mule in 1779?

F Greater efficiency in textile manufacturing

G An increase in the amount of cloth imported from Asia

H A decrease in the number of factories

J An increase in the price of finished goods

This question is asking you about the effect of the invention of the spinning mule.On the lines below, briefly write about the effects of such technologicaldevelopments as the invention of the spinning mule. If you don’t remember theeffects of the spinning mule, think about the effects of other inventions such as thecotton gin and the steam engine.

Remember to eliminate answer choices that did not result from the invention of thespinning mule. Consider answer choice (F). Did greater efficiency in textile manufac-turing occur as a result of the invention of the spinning mule? If yes, keep answerchoice (F).

Consider answer choice (G). Was more cloth imported from Asia as a result of theinvention of the spinning mule? If no, eliminate answer choice (G).

Consider answer choice (H). Was there a decrease in the number of factories as aresult of the invention of the spinning mule? If no, eliminate answer choice (H).

Consider answer choice (J). Was there an increase in the price of finished goods asa result of the invention of the spinning mule? If no, eliminate answer choice (J).

Look at the remaining answer choices. Based on what you wrote in step 1, choosethe most logical answer choice and briefly explain why you chose it.

Step6

Step5

Step4

Step3

Step2

Step1

Lesson 3: Relating Cause and Effect

36 TAKS Test Practice Workbook, Social Studies Grade 10

Page 41: TAKS Test Preparation Workbook, Grade 10 - Student Edition€¦ · Rights was created to address colonists’ anger at the British government for— F placing taxes on common goods

Cop

yrig

ht ©

by T

he M

cGra

w-H

ill C

ompa

nies

, In

c.

Lessons 37

Use ItLesson 3: Relating Cause and Effect

DIRECTIONS

Read each question carefully and then choosethe best answer.

1 The Emancipation Proclamation of 1862led to—

A freedom for slaves in both theSouthern and Northern United States

B greater support for the Union wareffort from Northern antislaverygroups

C European support for the Confederatewar effort

D the decision of many Confederatestates to reenter the Union

2 The Bill of Rights was added to theConstitution in 1791 in order to—

F explain the framework for the threebranches of the U.S. government

G state the contract between the peopleand the government

H limit the power of the nationalgovernment and guarantee the rightsof individuals

J restrict the rights of individuals bynarrowly defining them

3 Thousands of people rushed to Californiaduring the 1840s and 1850s because—

A of the discovery of gold there

B slavery was prohibited there

C its climate was mild

D it offered fertile land for farming

4 The main reason that Americans allowedenslaved African Americans to enlist inthe army during the Revolutionary Warwas that—

F Americans wanted to create a moreequitable society following the endof the war

G Americans were in desperate need oftroops and realized they were losingpotential soldiers

H the British were offering freedom toenslaved African Americans whoenlisted with them

J Crispus Attucks’s bravery andheroism were proven during theBoston Massacre

5 The Temperance Movement in themid-1800s resulted in—

A the creation of a colony in westernAfrica for free African Americans

B increased tolerance for prisoners andmentally ill people

C the founding of the Church of JesusChrist of Latter-day Saints

D some states limiting or prohibitingthe sale of alcohol

Page 42: TAKS Test Preparation Workbook, Grade 10 - Student Edition€¦ · Rights was created to address colonists’ anger at the British government for— F placing taxes on common goods

38 TAKS Test Practice Workbook, Social Studies Grade 10

Copyright ©

by The M

cGraw

-Hill C

ompanies, Inc.

6 President Jefferson declared a tradeembargo with France and Great Britainin 1807 in order to—

F foster trade with countries other thanFrance and Great Britain

G increase his popularity amongnortheastern ship owners andmerchants

H keep the United States out of theNapoleonic Wars and preventinterference with American trade

J encourage industry to producemore goods, making Americamore self-reliant

7 What was one of the effects of the BankPanic of 1837?

A The federal government began storingits funds in the federal treasury.

B People traded their gold and silvercoins for banknotes.

C Demand for factory goods increased,and unemployment fell.

D Land prices increased dramatically.

8 As a result of the invention of newmachines such as the cotton gin, thespinning jenny, and steam-powered loomand increased efficiency in the textileindustry—

F prices for manufactured goods rose

G cities began growing

H the government grew moreconservative

J the population shifted to morerural areas

Lesson 3: Relating Cause and Effect

Page 43: TAKS Test Preparation Workbook, Grade 10 - Student Edition€¦ · Rights was created to address colonists’ anger at the British government for— F placing taxes on common goods

Lessons 39

Learn ItSome questions on the TAKS may ask you to examine a sequence of events.You may need to determine the order in which certain events occurred,choose an event that occurred in sequence with several other events, ordecide which event out of a list occurred first or last. Learn how to answerthese questions by completing the following practice questions.

1� Which of the following events took place first?

A America’s first political parties emerged.

B George Washington was elected president.

C The Constitution was ratified.

D The Articles of Confederation were ratified.

This question tests your knowledge of the founding of the United States.On the lines below, briefly review the main events of this period.

Now look at all of the answer choices. Some events maybe the result of other events in the answer choices. Arethere any you can eliminate immediately? If so, list themand explain why those answer choices are incorrect.

Look at the remaining answer choices. Which oneprobably occurred first? Based on what you wrote insteps 1 and 2, choose the most logical answer choiceand briefly explain why you chose it.

Step3

Step2

Step1

Lesson

4

Test Tip

The process ofelimination can improveyour chances ofselecting the correctanswer. Even if youdon’t know the answerto a question, you canprobably eliminate oneor two answer choicesthat are definitely notcorrect.

Lessons 39

Cop

yrig

ht ©

by T

he M

cGra

w-H

ill C

ompa

nies

, In

c.

Putting Events into Sequence

Page 44: TAKS Test Preparation Workbook, Grade 10 - Student Edition€¦ · Rights was created to address colonists’ anger at the British government for— F placing taxes on common goods

Copyright ©

by The M

cGraw

-Hill C

ompanies, Inc.

Now try another question that asks you about a sequence of events.

2�

Based on the time line above, what inferences can you draw?

F The Northern states gained an increasing amount of political power before the Civil War.

G Enslaved African Americans increasingly gained rights before the Civil War.

H The issue of slavery became increasingly divisive prior to the Civil War.

J The government was becoming increasingly antislavery before the Civil War.

This question is testing your knowledge of the years leading up to the Civil War. Onthe lines below, briefly write what you know about this period.

Consider answer choice (F). Did the Northern states gain political power before theCivil War? If not, eliminate choice (F).

Consider answer choice (G). Did enslaved African Americans gain rights prior to theCivil War? If not, eliminate choice (G).

Consider answer choice (H). Was the issue of slavery becoming more divisivebefore the Civil War? If not, eliminate choice (H).

Consider answer choice (J). Was the government becoming increasingly antislaverybefore the Civil War? If not, eliminate choice (J).

Consider the remaining answer choices. Based on what you wrote in step 1,choose the most logical answer choice and briefly explain why you chose it.

Step6

Step5

Step4

Step3

Step2

Step1

Fugitive Slave Act

Southernstatessecede

1850

Kansas-Nebraska Act

1854

Dred Scottdecision

1857 1861

Lesson 4: Putting Events into Sequence

40 TAKS Test Practice Workbook, Social Studies Grade 10

Page 45: TAKS Test Preparation Workbook, Grade 10 - Student Edition€¦ · Rights was created to address colonists’ anger at the British government for— F placing taxes on common goods

Cop

yrig

ht ©

by T

he M

cGra

w-H

ill C

ompa

nies

, In

c.

Lessons 41

Use ItLesson 4: Putting Events into Sequence

DIRECTIONS

Read each question carefully and then choosethe best answer.

Use the information in the box and yourknowledge of social studies to answer question 1.

1 In which order did the presidents listedserve?

A 1, 2, 3, 4

B 4, 3, 2, 1

C 3, 2, 1, 4

D 3, 1, 2, 4

2 Which of the following events happenedmost recently?

F The Fugitive Slave Act imposedstiffer penalties on those helpingrunaway slaves.

G The Confederate States of Americawere formed.

H The Compromise of 1850 was signedto settle the issue of slavery in theterritories.

J The Supreme Court made the DredScott decision.

Use the information in the box and yourknowledge of social studies to answer question 3.

3 In which order were the above keyAmerican documents drafted?

A 1, 3, 2, 4

B 4, 3, 1, 2

C 4, 1, 3, 2

D 3, 1, 2, 4

1. U.S. Constitution

2. Bill of Rights

3. Declaration of Independence

4. Common Sense1. James Monroe

2. John Adams

3. George Washington

4. Abraham Lincoln

Page 46: TAKS Test Preparation Workbook, Grade 10 - Student Edition€¦ · Rights was created to address colonists’ anger at the British government for— F placing taxes on common goods

Use the time line and your knowledge of socialstudies to answer question 4.

4 Which of the following conclusions can beinferred from the time line?

F The United States began playing anincreasingly important role in globalpolitics.

G The United States needed greateramounts of natural resources tosupport the growing economy.

H The United States began extendingcitizenship rights to other people,including the French, Mexicans, andNative Americans.

J The United States gradually obtainedland from other countries and peoplesuntil it controlled the continent fromthe Atlantic to the Pacific Oceans.

Use the information in the box and yourknowledge of social studies to answer question 5.

5 In which order did the events listed occur?

A 1, 4, 2, 3

B 4, 2, 1, 3

C 3, 4, 2, 1

D 2, 3, 4, 1

6 Which event took place first?

F Colonists boycotted British goods.

G The colonies declared independence.

H The Stamp Act was passed.

J The First Continental Congress met.

7 Which of the following was invented first?

A Steam-powered train

B Telephone

C Sewing machine

D Steam-powered loom

Use the time line and your knowledge of socialstudies to answer question 8.

8 Based on the time line, what inference canyou draw?

F Colonists were becoming increasinglyupset by taxation withoutrepresentation.

G The colonies were moving toward thecreation of a strong union betweenstates.

H The relationships between colonistsand Native Americans were growinghostile.

J Colonists were growing reluctant toform a strong federal government.

Articles ofConfederation

passed

1754 1765 1774 1781 1788

The AlbanyPlan

The Stamp ActCongress meets

The FirstContinental Congress

meets

U.S.Constitution

ratified

1. The California Gold Rush

2. Industrial Revolution in the UnitedStates

3. The Civil War

4. American Revolution

LouisianaPurchase

Trailof Tears

Annexationof Texas

MexicanCession

1803 1838 1845 1848

Lesson 4: Putting Events into Sequence

42 TAKS Test Practice Workbook, Social Studies Grade 10

Copyright ©

by The M

cGraw

-Hill C

ompanies, Inc.

Page 47: TAKS Test Preparation Workbook, Grade 10 - Student Edition€¦ · Rights was created to address colonists’ anger at the British government for— F placing taxes on common goods

Lessons 43

Learn ItReading and Interpreting Maps

Some questions on the TAKS may ask you to use the information shown onmaps. Learn how to answer these questions by completing the followingpractice question.

1�

On the map, what do the gray shaded states have in common?

A They belonged to the Confederate States of America.

B They led the way in the Industrial Revolution.

C They were the original thirteen colonies.

D They were Union states where slavery was permitted.

This question tests your knowledge of U.S. history andgeography. Which states are shaded gray?

Step1

The United States, 1861

Test Tip

Some maps will beaccompanied by titlesand keys. Read all titlesand keys carefully. Theytell you what kind ofinformation is on themap.

Lessons 43

Cop

yrig

ht ©

by T

he M

cGra

w-H

ill C

ompa

nies

, In

c.

Lesson

5

Page 48: TAKS Test Preparation Workbook, Grade 10 - Student Edition€¦ · Rights was created to address colonists’ anger at the British government for— F placing taxes on common goods

44 TAKS Test Practice Workbook, Social Studies Grade 10

Copyright ©

by The M

cGraw

-Hill C

ompanies, Inc.

Think about where these states are located. Based on the map, what did the grayshaded areas have in common in 1861?

Consider all the answer choices. Are there any that can be eliminated immediatelybased on what you wrote in steps 1 and 2? If so, briefly explain why you think thosechoices are incorrect.

Look at the remaining answer choices. Based on what you wrote in step 3,choose the most logical answer choice and briefly explain why you chose it.

Step4

Step3

Step2

Lesson 5: Reading and Interpreting Maps

Page 49: TAKS Test Preparation Workbook, Grade 10 - Student Edition€¦ · Rights was created to address colonists’ anger at the British government for— F placing taxes on common goods

Cop

yrig

ht ©

by T

he M

cGra

w-H

ill C

ompa

nies

, In

c.

Lessons 45

Use ItLesson 5: Reading and Interpreting Maps

DIRECTIONS

Read each question carefully and then choosethe best answer.

Use the map and your knowledge of socialstudies to answer questions 1 and 2.

1 Which of the following conclusions canyou draw from the map of NorthernAfrica?

A Most Northern Africans depend onnomadic herding to make their living.

B Most Northern Africans raise foodand sell it to make a living.

C Most Northern Africans live in areaswhere there is little to do.

D Most Northern Africans are fisherswho make their living from the sea.

2 Which of the following conclusions doesthe map support?

F Morocco is the southernmost countryin North Africa.

G The Mediterranean Sea is south ofNorth Africa.

H Egypt is west of Libya.

J Tunisia is northeast of Algeria.

Use the map and your knowledge of socialstudies to answer questions 3 and 4.

3 In which area did most of theRevolutionary War take place?

A 4

B 3

C 2

D 1

4 Area 7 on the map is known as the—

F Oregon Territory

G Louisiana Purchase

H Gadsden Purchase

J Republic of Texas

The United States, 1853

5

6

7 4

31

2

Nomadic herdingCommercial farmingSubsistence farming

Commercial fishingLittle or no activity

AFRICA

AlgeriaWesternSahara

Libya Egypt

MediterraneanSea

TunisiaMorocco

Page 50: TAKS Test Preparation Workbook, Grade 10 - Student Edition€¦ · Rights was created to address colonists’ anger at the British government for— F placing taxes on common goods

46 TAKS Test Practice Workbook, Social Studies Grade 10

Copyright ©

by The M

cGraw

-Hill C

ompanies, Inc.

5 Which of the following conclusions aboutpopulation density in China does the mapsupport?

A Very few large cities are located nearthe coast.

B Eastern China is more denselypopulated than western China.

C Wuhan is the largest city in China.

D No large Chinese cities are locatednear rivers.

6 Which of the following can be concludedfrom the map?

F Shanghai is located in western China.

G Harbin is the southernmost city with apopulation of more than two million.

H Beijing is south of the Chang JiangRiver.

J Hong Kong is south of Shanghai,Wuhan, and Guangzhou.

Lesson 5: Reading and Interpreting Maps

Use the map and your knowledge of social studies to answer questions 5 and 6.

CHINA

MONGOLIA

East Asia: Population Density

Shanghai

Hong Kong

Xi’an

Tianjin

Qingdao

Wuhan

Nanjing

Guangzhou

Per sq. mi.Over 250126–25061–1252–60Under 2Uninhabited

Qiqhar

Shenyang

Changchun

Beijing

Kunming

Harbin

Lanzhou

Taiyuan

Chongqing

Page 51: TAKS Test Preparation Workbook, Grade 10 - Student Edition€¦ · Rights was created to address colonists’ anger at the British government for— F placing taxes on common goods

Lessons 47

Learn ItSome questions on the TAKS may ask you to draw conclusions from theinformation in charts. Learn how to answer these questions by completing thefollowing practice question.

1�

Which of the following best fits in the center circle?

A The Articles of Confederation

B The Executive branch

C Anti-Federalism

D Checks and balances

Look at the information in each of the circles.Consider what the information in each circle seems tobe describing and how the information in the circles isrelated. Consider answer choice (A). What are theArticles of Confederation?

Does the information in the diagram describe the Articles ofConfederation? If not, eliminate answer choice (A).

Step1

Presidenthas veto

power

Congress can override

a veto

The Senateapproves

judges

Congress can

impeachthe president

The courtcan declare

a lawunconstitutional

?

Lesson

6

Test Tip

Questions about chartsusually require someoutside knowledge toarrive at the correctanswer. If the answer tothe question is notdirectly stated in thechart, find the answerchoice that is supportedby what is directlystated in the chart.

Lessons 47

Cop

yrig

ht ©

by T

he M

cGra

w-H

ill C

ompa

nies

, In

c.

Reading and Interpreting Charts and Tables

Page 52: TAKS Test Preparation Workbook, Grade 10 - Student Edition€¦ · Rights was created to address colonists’ anger at the British government for— F placing taxes on common goods

48 TAKS Test Practice Workbook, Social Studies Grade 10

Copyright ©

by The M

cGraw

-Hill C

ompanies, Inc.

Consider answer choice (B). What is the executive branch?

Does the information in the diagram describe the executive branch? If not,eliminate answer choice (B).

Consider answer choice (C). What is Anti-Federalism?

Does the information in the diagram describe Anti-Federalism? If not, eliminateanswer choice (C).

Consider answer choice (D). What are checks and balances?

Does the information in the diagram describe checks and balances? If not,eliminate answer choice (D).

Look at the remaining answer choices. Based on both the details in the diagramand what you wrote in each step, choose the most logical answer choice and brieflyexplain why you chose it.

Step5

Step4

Step3

Step2

Lesson 6: Reading and Interpreting Charts and Tables

Page 53: TAKS Test Preparation Workbook, Grade 10 - Student Edition€¦ · Rights was created to address colonists’ anger at the British government for— F placing taxes on common goods

Cop

yrig

ht ©

by T

he M

cGra

w-H

ill C

ompa

nies

, In

c.

Lessons 49

Use ItLesson 6: Reading and Interpreting Charts and Tables

DIRECTIONS

Read each question carefully and then choosethe best answer.

Use the information in the box and yourknowledge of social studies to answer question 1.

1 Harriet Tubman and Frederick Douglassare best known for sharing the convictionthat—

A railways should be built across theentire United States

B alcohol should be prohibited exceptfor medical purposes

C public education should be subsidizedby the state

D enslaved African Americans shouldbe freed

Use the table and your knowledge of socialstudies to answer question 2.

2 Which conclusion can be drawn from thedata presented in the table?

F Tobacco production rose steadilybetween 1850 and 1880.

G Cotton production rose steadilybetween 1850 and 1880.

H Combined cotton and tobaccoproduction was highest in 1860.

J Tobacco production was higher in1870 than in 1850.

Agricultural Production in the South, 1850–1880

1850186018701880

2.85.83.25.9

9021085140

Cotton (mil. of bales)

Tobacco (mil. of pounds)Year

Nineteenth-Century Reformers

John Cocke Temperance Movement

Frederick Douglass Abolitionist Movement

Horace Mann Common SchoolMovement

Robert Owen Communal LivingMovement

Lucy Stone Women’s RightsMovement

Harriet Tubman Underground Railroad

Page 54: TAKS Test Preparation Workbook, Grade 10 - Student Edition€¦ · Rights was created to address colonists’ anger at the British government for— F placing taxes on common goods

50 TAKS Test Practice Workbook, Social Studies Grade 10

Copyright ©

by The M

cGraw

-Hill C

ompanies, Inc.

Use the information in the box and yourknowledge of social studies to answer question 3.

3 Which historical American document ischaracterized by the above statements?

A The U.S. Constitution

B The Articles of Confederation

C The Declaration of Independence

D The Bill of Rights

Use the diagram and your knowledge of socialstudies to answer question 4.

4 Which of the following phrases should beplaced in the center circle?

F The role of the president

G The role of the courts

H The duties of citizens

J The responsibilities of Congress

Use the table and your knowledge of socialstudies to answer questions 5 and 6.

5 Which statement is supported by the tableon the population of the United States?

A The United States declined in termsof geographic size between 1800and 1810.

B The United States saw the largestpercentage-of-population increasein 1840.

C The United States saw the largestpopulation increase between 1830and 1840.

D The 1790 census was incomplete andthe data were incorrect.

6 During which census year was the UnitedStates most densely populated?

F 1800

G 1820

H 1830

J 1840

Population of the United States

179018001810182018301840

3,929,2145,308,4837,239,8819,638,453

12,866,02017,069,453

4.56.14.35.57.49.8

Populationper square

mile

N/A35.136.433.133.532.7

Percentageincrease

sincepreviouscensus

Year Population

Obeylaws

Paytaxes

Attendschool

Sign upfor military

serviceServe on

juries

?

The federal government has the power todeclare war.

The federal government has the power tooversee relationships with foreigngovernments.

The federal government does not have thepower to regulate trade or collect taxes.

Lesson 6: Reading and Interpreting Charts and Tables

Page 55: TAKS Test Preparation Workbook, Grade 10 - Student Edition€¦ · Rights was created to address colonists’ anger at the British government for— F placing taxes on common goods

Lessons 51

Learn ItSome of the questions on the TAKS may ask you to draw conclusions frominformation in graphs. Learn how to answer these questions by completing thefollowing practice question.

1�

Which of the following is a valid conclusion based on the information inthe graph?

A Non-OPEC nations decreased their oil production in the 1980s.

B OPEC nations cooperate in determining the price of oil per barrel.

C OPEC nations produced more oil than non-OPEC nationsin 1970.

D Non-OPEC nations consume more crude oil thanthey produce.

Read the graph and become familiar with the kind ofinformation it contains. Consider answer choice (A). Isthere any information in the graph that indicates that non-OPEC nations decreased their oil production in the1980s? If not, eliminate answer choice (A). If so, describethe piece of data and explain how it helps you determinethat non-OPEC nations decreased their oil production inthe 1980s.

Step1

0

10

20

30

40

1960 1970 1980 1990

World Crude Oil Production, 1960–1990

OPEC countriesNon-OPEC countries

Mill

ion

s o

f B

arre

ls P

er D

ay

Lesson

7

Test Tip

Before you answer aquestion that isaccompanied by agraph, read the title ofthe graph and the labelson the x- and y-axes sothat you are familiarwith the information itcontains. Then use thatinformation to answerthe question.

Lessons 51

Cop

yrig

ht ©

by T

he M

cGra

w-H

ill C

ompa

nies

, In

c.

Reading and Interpreting Graphs

Page 56: TAKS Test Preparation Workbook, Grade 10 - Student Edition€¦ · Rights was created to address colonists’ anger at the British government for— F placing taxes on common goods

52 TAKS Test Practice Workbook, Social Studies Grade 10

Copyright ©

by The M

cGraw

-Hill C

ompanies, Inc.

Consider answer choice (B). Is there any information in the graph that relates to theprice of oil per barrel? If not, eliminate answer choice (B). If so, describe the pieceof data and explain how it helps you determine that OPEC nations cooperate todetermine oil prices.

Consider answer choice (C). Is there any information in the graph that indicates thatOPEC nations produced more oil than non-OPEC nations in 1970? If not, eliminateanswer choice (C). If so, describe the piece of data and explain how it helps youdetermine that OPEC nations produced more oil than non-OPEC nations in 1970.

Consider answer choice (D). Is there any information in the graph that relates tothe consumption of crude oil? If not, eliminate answer choice (D). If so, describethe piece of data and explain how it helps you determine that non-OPEC nationsconsume more oil than they produce.

Look at the remaining answer choices. Based on the data in the graph and whatyou wrote in steps 1 through 4, choose the most logical answer choice and brieflyexplain why you chose it.

Step5

Step4

Step3

Step2

Lesson 7: Reading and Interpreting Graphs

Page 57: TAKS Test Preparation Workbook, Grade 10 - Student Edition€¦ · Rights was created to address colonists’ anger at the British government for— F placing taxes on common goods

Cop

yrig

ht ©

by T

he M

cGra

w-H

ill C

ompa

nies

, In

c.

Lessons 53

Use ItLesson 7: Reading and Interpreting Graphs

1 From the graph you can conclude that—

A Rwanda is larger in area than Kenya

B the Democratic Republic of theCongo has the most fertile farmland

C South Africa has the fewestagricultural workers of all thecountries shown

D Zambia is wealthier than Zimbabwe

2 The graph best supports which of thefollowing conclusions?

F More people work in agriculture inTanzania than in the other countries.

G The number of people who graduatefrom college is highest in SouthAfrica.

H Botswana has a more diverseeconomy than most other Africannations.

J The percentage of people who do notwork in agriculture is nearly the samein Uganda and Zambia.

DIRECTIONS

Read each question carefully and then choose the best answer.

Use the graph and your knowledge of social studies to answer questions 1 and 2.

Agricultural Workforce(selected countries)

0 20 40 60 80 100

Botswana

KenyaRwanda

South AfricaTanzaniaUgandaZambia

Zimbabwe

DemocraticRepublic ofthe Congo

Percent

Co

un

trie

s

Page 58: TAKS Test Preparation Workbook, Grade 10 - Student Edition€¦ · Rights was created to address colonists’ anger at the British government for— F placing taxes on common goods

Use the graphs and your knowledge of socialstudies to answer question 3.

3 Which is the best inference based on thepie graphs?

A Manufacturing jobs dominated theU.S. economy in 1840.

B Manufacturing jobs grew more thanother jobs between 1840 and 1870.

C The number of agricultural andnonagricultural workers werealmost equal in 1870.

D People avoided working in factoriesduring the 1840s because of theterrible working conditions.

Use the graph and your knowledge of socialstudies to answer questions 4 and 5.

4 You can conclude from the line graph that—

F demand for American goods inforeign countries declined between1810 and 1820

G shipping to domestic ports rose moresteadily than shipping to foreign ports

H the number of people working inshipping was greatest in 1845

J a severe drought in 1830 led to adecline in shipping

5 Based on the graph, which of thefollowing is true?

A Shipping generated $600 million inprofits in 1820.

B More than 1,200 people wereemployed in shipping in 1825.

C Less than 400 tons of goods wereshipped in 1800.

D More than 2,000 tons of goods wereshipped in 1845.

0200400600800

1,0001,2001,400

1800

1805

1810

1815

1820

1825

1830

1835

1840

1845

American Merchant Shipping,1800–1845

Th

ou

san

ds

of

To

ns

Shipping to domestic portsShipping to foreign ports

16%

15%

69%

184020%

16%

64%

1850

23%18%

59%

186026% 21%

53%

1870

ManufacturingOther

Agricultural

Agricultural and NonagriculturalWorkers, 1840–1870

Lesson 7: Reading and Interpreting Graphs

54 TAKS Test Practice Workbook, Social Studies Grade 1054 TAKS Test Practice Workbook, Social Studies Grade 10

Copyright ©

by The M

cGraw

-Hill C

ompanies, Inc.

Page 59: TAKS Test Preparation Workbook, Grade 10 - Student Edition€¦ · Rights was created to address colonists’ anger at the British government for— F placing taxes on common goods

Lessons 55

Learn ItSome of the questions on the TAKS may ask you to interpret quotations. Learnhow to answer these questions by completing the following practice question.

1�

Which of the speakers plans to take advantage of the rights guaranteed by theFirst Amendment?

A Speaker 1

B Speaker 2

C Speaker 3

D Speaker 4

Summarize the First Amendment on the lines below.

Use your summary of the First Amendment to considereach of the answer choices.

Step1

Speaker 1: I think we should organize a rally to protest the loosening of environmentallaws. Congress should know that we care about our natural resources.

Speaker 2: It’s wrong to hold protests. You should trust our leaders to take the right action.

Speaker 3: It’s acceptable to express your opinions as long as you’re part of the majority.

Speaker 4: We have the right to express our beliefs, but only if we write letters to the editor.

Lesson

8

Test Tip

For questionsaccompanied byhistorical quotations,summarize the text ofthe quotations in yourown words before youanswer the question.Doing so will help youidentify the correctanswer more easily.

Lessons 55

Cop

yrig

ht ©

by T

he M

cGra

w-H

ill C

ompa

nies

, In

c.

Interpreting Quotations

Page 60: TAKS Test Preparation Workbook, Grade 10 - Student Edition€¦ · Rights was created to address colonists’ anger at the British government for— F placing taxes on common goods

56 TAKS Test Practice Workbook, Social Studies Grade 10

Copyright ©

by The M

cGraw

-Hill C

ompanies, Inc.

Consider answer choice (A). Does the First Amendment guarantee people’s rightsto hold rallies to protest government policies? If not, eliminate answer choice (A).If so, which part of the First Amendment refers to political rallies?

Consider answer choice (B). Does the First Amendment say that it’s wrong to holdprotests? If not, eliminate answer choice (B). If so, which part of the First Amendmentrefers to the unacceptability of holding protests?

Consider answer choice (C). Does the First Amendment indicate that it’s acceptableto express your opinions only if you’re part of the majority? If not, eliminate answerchoice (C). If so, which part of the First Amendment refers to the necessity of beingpart of the majority?

Consider answer choice (D). Does the First Amendment state that you may expressyour beliefs only in writing? If not, eliminate answer choice (D). If so, which part ofthe First Amendment requires that you express your beliefs only in writing?

Look at the remaining answer choices. Choose the answer choice that best reflectsthe First Amendment. Briefly explain why you chose this answer.

Step6

Step5

Step4

Step3

Step2

Lesson 8: Interpreting Quotations

Page 61: TAKS Test Preparation Workbook, Grade 10 - Student Edition€¦ · Rights was created to address colonists’ anger at the British government for— F placing taxes on common goods

Lessons 57

Use ItDIRECTIONS

Read each question carefully and then choosethe best answer.

Use the information in the box and yourknowledge of social studies to answer question 1.

1 Which speaker is describing one of ourresponsibilities as citizens of the UnitedStates?

A Speaker 1

B Speaker 2

C Speaker 3

D Speaker 4

Use the quotation and your knowledge ofsocial studies to answer question 2.

2 The woman described in this quotation ismost likely—

F leading Lewis and Clark on theirexpedition

G helping enslaved people escape viathe Underground Railroad

H traveling west on the Oregon Trail

J spying for the Confederate Armyduring the Civil War

“[My mother] was the oldest of sixchildren, and as there were some loosehorses and cattle every day which wouldnot follow the trail unless made to do so,she was required to ‘trail’ behind them andsee that none was lost. To be sure, thedistance made would not average morethan ten or twelve miles a day, but itnecessitated walking in the dust caused byhundreds of tramping oxen and horses,besides the duty of keeping the stubborn orcontrary or indifferent animals fromlagging behind.”

Speaker 1: By law, it’s one of our dutiesas citizens to vote, which iswhy there’s such a high voterturnout.

Speaker 2: My one vote does not make adifference.

Speaker 3: Because our leaders never dowhat we want, it seems likevoting is a waste of time.

Speaker 4: It’s important for everyone tovote in elections. If you donot vote, you are not reallyplaying a part in our society.

Lesson 8: Interpreting Quotations C

opyr

ight

©by

The

McG

raw

-Hill

Com

pani

es,

Inc.

Page 62: TAKS Test Preparation Workbook, Grade 10 - Student Edition€¦ · Rights was created to address colonists’ anger at the British government for— F placing taxes on common goods

58 TAKS Test Practice Workbook, Social Studies Grade 10

Copyright ©

by The M

cGraw

-Hill C

ompanies, Inc.

Use the excerpt and your knowledge of socialstudies to answer question 3.

3 The purpose of Sojourner Truth’s speechwas to—

A prove that African American womenwere stronger than white women

B ensure that the women’s rightsmovement included African Americanwomen in their fight for equality

C increase support for women whoworked as hired laborers on farms

D make free women aware of the poortreatment of enslaved AfricanAmerican women

Use the information in the box and yourknowledge of social studies to answer question 4.

4 The delegates to the ConstitutionalConvention addressed the colonialcomplaint above about King George’slimitless power with respect to the courtsystem by—

F making the national governmentstronger

G designing a system of checks andbalances

H dividing the government into threeseparate branches

J giving the power to appoint judges toCongress

[King George III] has made Judges dependenton his Will alone, for the tenure of their offices,and the amount and payment of their salaries.

—Declaration of Independence

“That man . . . says that women need to behelped into carriages, and lifted over ditches,and to have the best places everywhere. Nobody ever helps me into carriages, or overmud puddles, or gives me any best place,and ain’t I a woman? I have plowed, andplanted, and gathered into barns, and noman could head me—and ain’t I a woman?I could work as much and eat as much as aman (when I could get it), and bear the lashas well—and ain’t I a woman? I have bornethirteen children and seen them most all soldoff into slavery, and when I cried out with amother’s grief, none but Jesus heard—andain’t I a woman?”

—Adapted excerpt from a speech thatSojourner Truth gave at a Woman’s Rights

Convention in 1851

Lesson 8: Interpreting Quotations

Page 63: TAKS Test Preparation Workbook, Grade 10 - Student Edition€¦ · Rights was created to address colonists’ anger at the British government for— F placing taxes on common goods

Some questions on the TAKS may ask you to read and interpret a cartoon oran illustration. Learn how to answer these types of questions by completing thefollowing practice question.

1�

This cartoon illustrates the point of view that—

A British rule was causing the colonies to turn against each other

B the colonies needed to unite to achieve independence from Great Britain

C civil war would destroy the American way of life

D the United States needed to expand for economic reasons

Look at the pieces of the snake. What do they represent?Step

1

Circa 1754

Lesson

9

Test Tip

Take some time tostudy the illustrationbefore reading thequestions about it. Ifyou understand theillustration before youtry to answer thequestions, you will findthe correct answermore easily.

Cop

yrig

ht ©

by T

he M

cGra

w-H

ill C

ompa

nies

, In

c.

Lessons 59

Learn ItInterpreting Illustrations and Political Cartoons

Page 64: TAKS Test Preparation Workbook, Grade 10 - Student Edition€¦ · Rights was created to address colonists’ anger at the British government for— F placing taxes on common goods

60 TAKS Test Practice Workbook, Social Studies Grade 10

Copyright ©

by The M

cGraw

-Hill C

ompanies, Inc.

Look at the text. What does it suggest about the illustration?

Look at the date the cartoon appeared. What was happening during this time in thecolonies?

Read the answer choices one at a time. After you have read an answer choice,look at the illustration. Ask yourself, “Is there any information in the illustration tosupport this answer choice?” Eliminate answer choices that are not supported byinformation in the illustration. List the choices you eliminated on the lines below.Explain your reasons for eliminating these answer choices.

Look at the answer choices you have not eliminated. If only one choice remains,your work is done! Record your answer choice, and then move on to the nextquestion. If more than one choice remains, look at the remaining answer choices.Which is most likely correct, and why?

The correct answer is .

Step5

Step4

Step3

Step2

Lesson 9: Interpreting Illustrations and Political Cartoons

Page 65: TAKS Test Preparation Workbook, Grade 10 - Student Edition€¦ · Rights was created to address colonists’ anger at the British government for— F placing taxes on common goods

Cop

yrig

ht ©

by T

he M

cGra

w-H

ill C

ompa

nies

, In

c.

Lessons 61

Use ItLesson 9: Interpreting Illustrations and Political Cartoons

1 What does this illustration depict?

A The construction of the U.S. Capitolafter unanimous state approval of theConstitution

B The rejection of the Constitution byVirginia and New York, and thedissolution of the Union

C The ratification of the Constitution bytwo powerful states from whom thegovernment needed support

D The long, arduous process of draftingthe U.S. Constitution at theconvention in Philadelphia

2 Based on the illustration, which state wasthe first to ratify the new Constitution?

F New Jersey

G New York

H Virginia

J Delaware

DIRECTIONS:

Read each question carefully and then choose the best answer.

Use the illustration and your knowledge of social studies to answer questions 1 and 2.

The Ninth PILLAR erected !“The Ratification of the Conventions of nine States, shall be sufficient for the establishment

of this Constitution, between the States so ratifying the same.” Art.vii

If it is not upit will rise.

The Attraction mustbe irresistible.

DE

L.

PE

N.

N.J

ER

.

GE

OR

.

CO

N.

MA

SS

A.

MA

RY.

S. C

AR

O.

N.H

AM

P.VIR

G.

N.YORK

Page 66: TAKS Test Preparation Workbook, Grade 10 - Student Edition€¦ · Rights was created to address colonists’ anger at the British government for— F placing taxes on common goods

Copyright ©

by The M

cGraw

-Hill C

ompanies, Inc.

3 The sentiment expressed by this cartoonis that—

A Revolutionary soldiers should begiven preferential treatment

B the colonists are tired of housingBritish troops

C it is futile for the British to try todefeat the colonists

D the war should end before colonistssuffer more losses

4 The political cartoon uses the term“Burgoyn’d.” Burgoyne was a Britishgeneral who was defeated by the colonists.What does the cartoon mean when it saysthere is “room for more . . . ”?

F The colonial armies were prepared todefeat additional British troops.

G The colonists were willing to livepeacefully with the British.

H The colonists had new jails toimprison other British troops.

J The colonists were anxious abouttheir ability to win other key battles.

Lesson 9: Interpreting Illustrations and Political Cartoons

Use the cartoon and your knowledge of social studies to answer questions 3 and 4.

THE AMERICAN RATTLESNAKE

Two British ArmiesI have thus Burgoyn’d,

And room for moreI’ve got behind.

62 TAKS Test Practice Workbook, Social Studies Grade 10

Page 67: TAKS Test Preparation Workbook, Grade 10 - Student Edition€¦ · Rights was created to address colonists’ anger at the British government for— F placing taxes on common goods
Page 68: TAKS Test Preparation Workbook, Grade 10 - Student Edition€¦ · Rights was created to address colonists’ anger at the British government for— F placing taxes on common goods
Page 69: TAKS Test Preparation Workbook, Grade 10 - Student Edition€¦ · Rights was created to address colonists’ anger at the British government for— F placing taxes on common goods

DIRECTIONS

Read each question and choose the bestanswer. Then mark the answer you havechosen.

SAMPLE B

Which of these inventions had the greatestimpact on the development of NewOrleans in the 1800s?

A Steamship

B Telegraph

C Steel plow

D Cotton gin

Cop

yrig

ht ©

by T

he M

cGra

w-H

ill C

ompa

nies

, In

c.

Practice Test 65

Page 70: TAKS Test Preparation Workbook, Grade 10 - Student Edition€¦ · Rights was created to address colonists’ anger at the British government for— F placing taxes on common goods

1 The northeastern United States became anindustrial center during the late 1700s andearly 1800s because of—

A the abundance of powerful rivers

B its well-developed ports

C its proximity to mineral resources

D its highly educated population

2 Which of the following contributed to thecolonists’ desire to have a representativegovernment?

F The French Revolution

G Slavery

H Loyalty to the British king

J Town meetings

3 1787 was a significant year in Americanhistory because—

A the colonies declared theirindependence from Great Britain

B Britain and France negotiated the endof the French and Indian War

C the Founders gathered in Philadelphiafor the Constitutional Convention

D Lincoln delivered the GettysburgAddress to bring the North and Southtogether

Use the information in the box andyour knowledge of social studies to answer question 4.

4 In which order were the documents listedabove created?

F A, B, C, D

G D, B, A, C

H D, C, A, B

J B, A, D, C

A. U.S. Constitution

B. Articles of Confederation

C. U.S. Bill of Rights

D. English Bill of Rights

66 TAKS Test Practice Workbook, Social Studies Grade 10

Copyright ©

by The M

cGraw

-Hill C

ompanies, Inc.

Page 71: TAKS Test Preparation Workbook, Grade 10 - Student Edition€¦ · Rights was created to address colonists’ anger at the British government for— F placing taxes on common goods

Use the quotation and your knowledge ofsocial studies to answer question 5.

5 In the quotation, the speaker expresses—

A support for the United States’sexpansion efforts

B gratitude for the military might of theUnited States

C relief about the removal of NativeAmericans from their land

D criticism of some Americans’ belief inManifest Destiny

6 As a result of the Glorious Revolution,which brought an end to King James II’sreign and the Dominion of New England,colonists gained the right to—

F export raw goods to any country

G have their own elected assemblies

H ignore English laws

J send representatives to England’sparliament

7 Which principle in the U.S. Constitutionwas specifically designed to protectcitizens against a tyrannical ruler?

A Federalism

B Checks and balances

C Limited government

D Popular sovereignty

Use the quotation and your knowledge ofsocial studies to answer question 8.

8 During approximately which year didPresident Lincoln make these remarks?

F 1776

G 1812

H 1850

J 1862

“If I could save the Unionwithout freeing any slave,I would do it; and if I couldsave it by freeing all the slaves,I would do it.”

—Abraham Lincoln

“The North Americans haveconquered whatever territoryadjoins them. In less than halfa century, they have becomemasters of extensive colonieswhich formerly belonged toSpain and France and of . . .territories from which havedisappeared the former owners,the Indian tribes.”

—Manuel Mier y Teran,Mexican general, 1830

Cop

yrig

ht ©

by T

he M

cGra

w-H

ill C

ompa

nies

, In

c.

Practice Test 67

Page 72: TAKS Test Preparation Workbook, Grade 10 - Student Edition€¦ · Rights was created to address colonists’ anger at the British government for— F placing taxes on common goods

9 Traditional subsistence farmers grow justenough food to feed their families, whilecommercial farmers are more likely toproduce—

A foodstuffs that can be preserved

B manufactured items

C import goods

D cash crops

Use the information in the box and yourknowledge of social studies to answer question 10.

10 The statements above describe which kindof economic system?

F A communist system

G A capitalist system

H A command economy

J A socialist system

11 From 1840 to 1860, tens of thousandsof people streamed west toward OregonCountry in search of—

A fertile farmland

B gold

C freedom from religious oppression

D higher-paying jobs

1. The government does little tointerfere with the operationof businesses.

2. Businesses are funded withprivate capital and areprivately owned.

3. The production of goods isbased upon demand for them.

68 TAKS Test Practice Workbook, Social Studies Grade 10

Copyright ©

by The M

cGraw

-Hill C

ompanies, Inc.

Page 73: TAKS Test Preparation Workbook, Grade 10 - Student Edition€¦ · Rights was created to address colonists’ anger at the British government for— F placing taxes on common goods

Use the graph and your knowledge of socialstudies to answer question 12.

12 During which five-year period was therethe most dramatic increase inimmigration?

F 1860–1865

G 1875–1880

H 1885–1890

J 1890–1895

Use the information in the box and yourknowledge of social studies to answer question 13.

13 Based on the list above, which of thefollowing conclusions could be drawnconcerning the Columbian Exchange?

A Citrus fruits are now grown onlyin North America.

B Europeans brought beans fromSpain into Africa.

C Peanuts were brought from theAmericas to Europe.

D Honeybees were brought from theAmericas to Europe.

Immigration, 1860–1900

050

100150200250300350400450500

1865

1860

1870

1875

1880

1885

1890

1895

1900

Imm

igra

nts

(In

th

ou

san

ds)

Cop

yrig

ht ©

by T

he M

cGra

w-H

ill C

ompa

nies

, In

c.

Practice Test 69

Potatoes

Pumpkins

Beans

Quinine

Tobacco

Peanuts

Turkeys

Disease

Peppers

Corn

Avocados

Sugarcane

Grains

Coffee beans

Honeybees

Bananas

Livestock

Turnips

Disease

Citrus fruits

Olives

Onions

The Columbian Exchange: ExchangeBetween the Americas and Europe,

Asia, and Africa

From Americas From Europe,Africa, and Asia

Page 74: TAKS Test Preparation Workbook, Grade 10 - Student Edition€¦ · Rights was created to address colonists’ anger at the British government for— F placing taxes on common goods

70 TAKS Test Practice Workbook, Social Studies Grade 10

Copyright ©

by The M

cGraw

-Hill C

ompanies, Inc.

14 According to the chart, which of thefollowing conclusions is true?

F Egypt lacks natural resources.

G Israel is more highly developedthan Egypt.

H Egypt has a more highly skilled laborforce than Iraq.

J Israel has a bigger populationthan Iraq.

Use the chart and your knowledge of social studies to answer question 14.

Comparative Data on Middle Eastern Countries

Country LiteracyRate

Annual OilProduction

MajorImports

MajorExports

Iraq 58% 58.3m MachineryVehicles

FuelsEnergy

Israel 95% 250m inreserves

Consumergoods

Machinery Chemicals

Egypt 51% 43.9m FoodstuffsMachinery

Petroleum productsCotton products

Fuels

Page 75: TAKS Test Preparation Workbook, Grade 10 - Student Edition€¦ · Rights was created to address colonists’ anger at the British government for— F placing taxes on common goods

15 Which of the following events took placein 1776?

A The Continental Army won a keyvictory against the British naval fleet.

B George Washington becamecommander of the Continental Army.

C The Second Continental Congressdrafted the Olive Branch Petition totry to avoid war with Britain.

D The Second Continental Congressissued the Declaration ofIndependence.

Use the map and your knowledge of socialstudies to answer question 16.

16 Using the map, what can you infer aboutthe location of cities in South America?

F Most cities are located in the northernpart of the continent.

G Most cities are located alongthe coast.

H Most cities are located in Colombia.

J Most cities are located in the interiorof the continent.

Caracas

Brasília

BelémFortaleza

RecifeSalvador

Rio deJaneiro

São Paulo

CuritibaPorto Alegre

Maracaibo

CaliQuito

Lima

La Paz

BogotáAtlanticOcean

AtlanticOcean

PacificOcean

MontevideoBuenos Aires

Rosario

ValaparaísoSantiago

Over 10,000,0005,000,000–10,000,0002,000,000–5,000,0001,000,000–2,000,000250,000–1,000,000

Guayaquil

Population Densityof Cities

Asuncion

Medellín

Cop

yrig

ht ©

by T

he M

cGra

w-H

ill C

ompa

nies

, In

c.

Practice Test 71

Page 76: TAKS Test Preparation Workbook, Grade 10 - Student Edition€¦ · Rights was created to address colonists’ anger at the British government for— F placing taxes on common goods

17 The “Declaration of Natural Rights”section of the Declaration of Independencemost likely guided colonial leadersas they—

A created a system of checks andbalances

B drafted the Bill of Rights

C divided power between federal andstate governments

D developed the electoral college system

18 In a market economy, the production ofgoods is based on demand for them. Incontrast, in a command economy, theproduction of goods is based on—

F the availability of natural resources

G consumer demand

H government decisions

J balance of trade

19 Which of the following resulted from thecreation of a system of makinginterchangeable parts?

A Less production of goods

B Equal distribution of wealth

C Higher prices for goods

D Increased demand for factory workers

20 One of the ways the U.S. Constitutionaddressed colonial grievances with GreatBritain was by—

F granting more power to the states thanto the federal government

G creating a representative government

H abolishing all taxes

J discouraging immigration to theUnited States

72 TAKS Test Practice Workbook, Social Studies Grade 10

Copyright ©

by The M

cGraw

-Hill C

ompanies, Inc.

Page 77: TAKS Test Preparation Workbook, Grade 10 - Student Edition€¦ · Rights was created to address colonists’ anger at the British government for— F placing taxes on common goods

21 The 1793 document issued by GeorgeWashington, which declared that theUnited States would be “friendly andimpartial” during the French Revolution,was called the—

A Declaration of Independence

B Emancipation Proclamation

C New Deal

D Proclamation of Neutrality

22 The number of Native American peopleswho lived in New England during the earlycolonial period declined dramatically forwhich of the following reasons?

F The Native Americans were afraid ofthe settlers and moved away.

G Native Americans were quicklyabsorbed into the colonists’communities, leaving behind theircultures.

H The region had experienced years ofsevere weather that destroyed NativeAmerican crops.

J Colonists’ farms ruined the habitat forwild game that Native Americans usedfor food.

Use the information in the box and yourknowledge of social studies to answer question 23.

23 What would be a good title for this listof items?

A Rights Guaranteed by the Declarationof Independence

B Rights Guaranteed by the Bill ofRights

C Articles of the Constitution

D Articles of Confederation

1. Religious freedom

2. No unauthorizedsearches

3. Speedy, fair trials

4. Ability to hold protests

5. Privacy at home

Cop

yrig

ht ©

by T

he M

cGra

w-H

ill C

ompa

nies

, In

c.

Practice Test 73

Page 78: TAKS Test Preparation Workbook, Grade 10 - Student Edition€¦ · Rights was created to address colonists’ anger at the British government for— F placing taxes on common goods

24 Which of the following is an accurateconclusion about the states listed in thechart?

F Massachusetts manufactures moregoods than any other state.

G Of the five states listed, New Jerseyhad the most representation in theHouse of Representatives in 1990.

H New Hampshire is the smallest stateon the eastern seaboard.

J North Carolina was the last stateadmitted to the Union.

74 TAKS Test Practice Workbook, Social Studies Grade 10

Copyright ©

by The M

cGraw

-Hill C

ompanies, Inc.

Use the chart and your knowledge of social studies to answer question 24.

New JerseyMassachusettsNew HampshireNorth Carolina

Kentucky

7,4187,8388,96948,71839,732

Land Area(sq. miles)

17871788178817891792

Year Admittedto UnionState

7,730,1886,016,4251,109,2526,628,6373,685,296

Population(1990)

Page 79: TAKS Test Preparation Workbook, Grade 10 - Student Edition€¦ · Rights was created to address colonists’ anger at the British government for— F placing taxes on common goods

25 Based on the map, which of the followingconclusions can you draw?

A New York had the largest statepopulation, which is why it had thegreatest number of colleges anduniversities.

B The literacy rate was lower in theSouth than in the North.

C Westerners placed less value onhigher education.

D Residents of the Northeast had moreeducational opportunities than the restof the country.

Cop

yrig

ht ©

by T

he M

cGra

w-H

ill C

ompa

nies

, In

c.

Practice Test 75

Use the map and your knowledge of social studies to answer question 25.

Wash.

Idaho

Oregon

Calif.

NevadaWyoming

Montana

S.D.

Iowa

Minn.

Ind.2

Ohio6

Ky. 4

Tenn. 2

Ala. 1

Ga. 3

S.C. 3

N.C.2

Fla.

W.Va. Va. 5

Pa.14

N.Y.13

Ill.4

Wis.

Mo.2

N.D.

Arizona

Texas

Oklahoma Ark.

La.1

Mich.1

Miss.1

ColoradoKansas

Nebraska

NewMexico

Utah N.J. 3Del.

Conn. 2R.I. 4Mass. 7

Md. 6

Wash., D.C. 2

Vt. 5N.H. 1

Colleges and Universities Founded Before 1830

Maine3

Page 80: TAKS Test Preparation Workbook, Grade 10 - Student Edition€¦ · Rights was created to address colonists’ anger at the British government for— F placing taxes on common goods

26 One of the most important factors thatdirectly contributed to the outbreak of theCivil War was—

F the existence of the UndergroundRailroad, which helped enslavedpersons escape

G conflict over whether slavery shouldbe legal in western states

H the passage of the Homestead Act,which gave free land to settlers

J strict laws that imposed stiff penaltiesfor aiding enslaved people who hadrun away

27 The invention of the telegraph, a devicethat allowed messages to be sent across awire, led to—

A a more well-informed populacethroughout the United States

B longer telephone conversationsbetween far-flung relatives

C public outcry about the miles of wirestrung between cities

D the end of the Pony Express and theU.S. Postal Service

Use the information in the box and yourknowledge of social studies to answer question 28.

28 Which amendment protects thedemonstrators’ right to act as describedabove?

F The First Amendment

G The Second Amendment

H The Third Amendment

J The Fifth Amendment

Approximately 1,000demonstrators held a rallyin front of the White Houseyesterday to protest thepresident’s unwillingness totighten gun laws. A recent slewof shootings at schools hasbrought together many differentgroups who are publiclycriticizing the ease with whichpeople can buy weapons.

76 TAKS Test Practice Workbook, Social Studies Grade 10

Copyright ©

by The M

cGraw

-Hill C

ompanies, Inc.

Page 81: TAKS Test Preparation Workbook, Grade 10 - Student Edition€¦ · Rights was created to address colonists’ anger at the British government for— F placing taxes on common goods

29 Which event on the time line most directlyled to the drafting of the U.S. Constitution?

A Annapolis Convention

B British surrender at Yorktown

C Boston Tea Party

D Declaration of Independence

Cop

yrig

ht ©

by T

he M

cGra

w-H

ill C

ompa

nies

, In

c.

Practice Test 77

Use the time line and your knowledge of social studies to answer question 29.

1773 1776 1781 1786 1787

BostonTea Party

Declaration ofIndependence

Britishsurrender at Yorktown

AnnapolisConvention

Shays’sRebellion

Page 82: TAKS Test Preparation Workbook, Grade 10 - Student Edition€¦ · Rights was created to address colonists’ anger at the British government for— F placing taxes on common goods

30 Based on the map, which of the followingis true?

F There was a direct line fromChattanooga to Montgomery.

G The railroad did not extend west ofthe Missouri River.

H The farthest east the railroad extendedwas to Kansas City.

J La Crosse was the northernmost citythat could be reached by train.

78 TAKS Test Practice Workbook, Social Studies Grade 10

Copyright ©

by The M

cGraw

-Hill C

ompanies, Inc.

Use the map and your knowledge of social studies to answer question 30.

Major Railroads, 1860

Railroad line

Kansas CitySt. Louis

New Orleans

Cairo

Tampa

Jacksonville

Charleston

Savannah

Atlanta

BostonLa CrosseDetroit

MadisonChicago

St. JosephQuincy

Memphis

Vicksburg Montgomery

Jackson

Chatanooga

Louisville

CincinnatiLynchburg

Toledo

Albany

Montpelier

New York City

Philadelphia

Richmond

BaltimoreWashington, D.C.

Mis

siss

ippi

R.

Missouri R.

Jackson

ClevelandPittsburgh

Page 83: TAKS Test Preparation Workbook, Grade 10 - Student Edition€¦ · Rights was created to address colonists’ anger at the British government for— F placing taxes on common goods

31 During the mid-1800s, the most significanteffect of the development of steamlocomotives was to—

A increase the speed at which goodscould be transported

B make travel throughout the UnitedStates more pleasant

C decrease the demand formanufactured goods

D help the United States become aninternational trade power

32 Which of the following statementsdescribes the major difference between theFourteenth and Fifteenth Amendments?

F The Fourteenth Amendment grantedvoting rights to formerly enslavedAfrican Americans, and the FifteenthAmendment granted voting rights toall citizens.

G The Fourteenth Amendment penalizedstates that did not extend voting rightsto African American men, while theFifteenth Amendment guaranteedvoting rights to African Americanmen.

H The Fourteenth Amendment endedslavery, and the Fifteenth Amendmentgranted all citizens over the age ofeighteen the right to vote.

J The Fourteenth Amendment reducedthe number of representatives thatSouthern states could send toCongress, while the FifteenthAmendment restored them.

Cop

yrig

ht ©

by T

he M

cGra

w-H

ill C

ompa

nies

, In

c.

Practice Test 79

Page 84: TAKS Test Preparation Workbook, Grade 10 - Student Edition€¦ · Rights was created to address colonists’ anger at the British government for— F placing taxes on common goods

33 Which of the following inventions belongsin the center of the diagram?

A Spinning jenny

B Steam-powered tools

C Cotton gin

D Sewing machine

80 TAKS Test Practice Workbook, Social Studies Grade 10

Copyright ©

by The M

cGraw

-Hill C

ompanies, Inc.

Use the diagram and your knowledge of social studies to answer question 33.

Faster processing

Increasedproduction

Industrialgrowth in

New England

Higherprices for

slaves

Greater demand for land

?

Page 85: TAKS Test Preparation Workbook, Grade 10 - Student Edition€¦ · Rights was created to address colonists’ anger at the British government for— F placing taxes on common goods

34 During the Revolutionary War, Patriotswere those who—

F wanted to break from British rule

G purchased goods produced only inAmerica

H remained loyal to the Britishmonarchy

J wrote the Declaration ofIndependence

Use the excerpt and your knowledge of socialstudies to answer question 35.

35 This excerpt demonstrates Thomas Paine’sbelief that—

A the colonists must declareindependence from Great Britain

B the easiest way for the coloniststo achieve independence wasthrough war

C the colonists should continuenegotiating with Parliament

D Great Britain would grant Americaindependence without conflict

36 States’ rights played a major role in theyears leading up to the Civil War forwhich of the following reasons?

F The issue of whether slavery shouldbe allowed in Southern states

G The controversy about extendingvoting rights to enslaved people

H The right of western states to vote onthe legality of slavery

J The debate over whether Southernstates should be allowed to secede

The period of debate is closed.Arms, as a last resort, must decidethe contest. . . . Everything that isright or reasonable pleads forseparation.

—Common Sense

Cop

yrig

ht ©

by T

he M

cGra

w-H

ill C

ompa

nies

, In

c.

Practice Test 81

Page 86: TAKS Test Preparation Workbook, Grade 10 - Student Edition€¦ · Rights was created to address colonists’ anger at the British government for— F placing taxes on common goods

37 Based on the data in the chart, which ofthe following conclusions is true?

A New Hampshire was founded by agroup of atheists.

B The majority of the New Englandcolonists was seeking religiousfreedom.

C Plymouth colonists were not involvedin fishing or farming.

D Later New England colonies were notfounded for religious reasons.

82 TAKS Test Practice Workbook, Social Studies Grade 10

Copyright ©

by The M

cGraw

-Hill C

ompanies, Inc.

Use the chart and your knowledge of social studies to answer question 37.

Plymouth

Mass. Bay Colony

Religious freedom

Religious freedom

Religious freedom

New England ColoniesColony Date Founded Reasons Founded Founders or Leaders

Massachusetts

New Hampshire

Rhode Island

Connecticut

1620

1630

1622

1636

1636

Profit from trade and fishing

Profit from fur trade and farmingReligious and political freedom

John Carver

William Bradford

John Winthrop

Ferdinando GorgesJohn Mason

Roger Williams

Thomas Hooker

Page 87: TAKS Test Preparation Workbook, Grade 10 - Student Edition€¦ · Rights was created to address colonists’ anger at the British government for— F placing taxes on common goods

Use the map and your knowledge of socialstudies to answer question 38.

38 Which of the following can you concludefrom the map?

F The Arabian Desert is located inJordan.

G The Suez Canal connects the Gulf ofSuez and the Mediterranean Sea.

H The Taurus Mountains are located inSyria.

J The Mediterranean Sea is located eastof Lebanon.

39 The invention of the machine that enabledworkers to make interchangeable parts andmass-produce goods led to—

A a decrease in the need for skilledworkers

B an increase in the price of factory-made goods

C an improvement in workingconditions at factories

D a decline in the number of childrenwho worked

40 Which of the following statements reflectsone of Thomas Jefferson’s beliefs?

F The U.S. government should includemore of the less-advantaged classes.

G The size of the army and navy shouldincrease.

H The government should control tradeand commerce.

J The federal government should havemore power.

SyriaLebanonIsrael

Egypt

Taurus Mtns.

Arabian

Desert

JordanSuezCanal

Gulf of S

uez

Med

iterranean

Sea

SaudiArabia

Iraq

An NafudDesert

Syrian

Desert

Cop

yrig

ht ©

by T

he M

cGra

w-H

ill C

ompa

nies

, In

c.

Practice Test 83